didi0504
Thanks Received: 0
Forum Guests
 
Posts: 23
Joined: October 20th, 2010
 
 
 

Q9 - A recent study of several

by didi0504 Tue Nov 09, 2010 2:33 am

Could you explain why is D wrong?

Thanks!
 
giladedelman
Thanks Received: 833
LSAT Geek
 
Posts: 619
Joined: April 04th, 2010
 
 
 

Re: Q9 - A recent study of several

by giladedelman Sun Nov 14, 2010 11:53 pm

Thanks for posting!

I'll explain why (D) is wrong, BUT, first, I want you to try explaining why you think it's right. How does it weaken the researchers' conclusion?
User avatar
 
noah
Thanks Received: 1192
Atticus Finch
Atticus Finch
 
Posts: 1541
Joined: February 11th, 2009
 
 
 

Re: PT 48 S4 Q9 - A recent study of several hundred female

by noah Mon Nov 15, 2010 12:36 pm

Another student posted this question:

Can someone explain to me why D isn't the answer?

If the physicians in the study had not been screened in advance to ensure that none had preexisting heart conditions, wouldn't that weaken the study? Thanks
 
itzakadoozie412
Thanks Received: 0
Vinny Gambini
Vinny Gambini
 
Posts: 17
Joined: September 13th, 2010
 
 
 

Re: PT 48 S4 Q9 - A recent study of several hundred female

by itzakadoozie412 Tue Jan 04, 2011 3:27 pm

Is it because (D) doesn't most weaken the argument like its asking us to. Maybe the physicians were screened after the research was done, which could still point to whether there's a or not the physicians had a preexisting heart condition. (A) weakens moreso than (D) becuase it implicating tha tother nutrients can alos be responsible for the inhibition of the heart disease, and not just B6/Folate. So does it weaken the author's correlation between B6/Folate and the inhibition of hear disease in women? I guess (D) doesnt do this
 
giladedelman
Thanks Received: 833
LSAT Geek
 
Posts: 619
Joined: April 04th, 2010
 
This post thanked 3 times.
 
 

Re: PT 48 S4 Q9 - A recent study of several hundred female

by giladedelman Thu Jan 06, 2011 4:39 pm

Remember that our job here is to weaken the argument, which really means undermining the conclusion.

So, what is the conclusion? It's that folate and B6 inhibit heart disease in women. And it's based on the premise that the women in this study showed an inverse relationship between coronary disease and dietary intake of folate and B6.

In other words, the study shows a correlation between lack of these vitamins and heart disease; from that correlation, the argument concludes that there's a causal relationship between the two factors. We can expect that the correct answer will weaken this argument by providing an alternate explanation of this correlation.

(A) is correct because it presents just such an alternate explanation: it's not the lack of B6 and folate that increases heart disease, it's the lack of other stuff which is found in the same foods as B6 and folate. So if you don't eat foods rich in B6 and folate, then you may have an increased risk of heart disease, but not because of the absence of those particular vitamins.

(D), meanwhile, just has no bearing on the question, "do folate and B6 inhibit heart disease in women?" Whether the women were screened for preexisting heart conditions is irrelevant; the premise tells us that there's an inverse correlation between heart disease and dietary intake of B6 and folate.

(B) is out of scope because the argument has nothing to do with men.

(C) is incorrect because health knowledge has no bearing on the argument.

(E) is incorrect because, okay, it's only present in a small number of foods, but does it inhibit heart disease or not?

Does that clear this one up for you guys?
 
socatkn1p
Thanks Received: 0
Forum Guests
 
Posts: 5
Joined: December 01st, 2010
 
 
 

Re: Q9 - A recent study of several hundred female

by socatkn1p Fri May 13, 2011 1:31 am

although (A) is the best answer, i dislike how this question is worded because the female physicians could have been taking supplements of folate and b6 along with foods that do not contain nonvitamin nutrients that inhibit heart disease.
 
giladedelman
Thanks Received: 833
LSAT Geek
 
Posts: 619
Joined: April 04th, 2010
 
 
 

Re: Q9 - A recent study of several hundred female

by giladedelman Sun May 15, 2011 5:24 pm

I suppose they could have, but what does that have to do with the argument?
 
goriano
Thanks Received: 12
Atticus Finch
Atticus Finch
 
Posts: 113
Joined: December 03rd, 2011
 
 
 

Re: Q9 - A recent study of several hundred female

by goriano Thu Jan 12, 2012 9:59 pm

giladedelman Wrote:I suppose they could have, but what does that have to do with the argument?


although (A) is the best answer, i dislike how this question is worded because the female physicians could have been taking supplements of folate and b6 along with foods that do not contain nonvitamin nutrients that inhibit heart disease.


I share socatkn1p's same concern with this question. The stimulus states that coronary heart disease was inversely proportional to their "dietary intake of folate and B6." To me, this means that the correlation was looking at folate and B6 IN ISOLATION, not folate, B6, plus whatever else in their diet. I think socatkn1p was addressing the possibility that the physicians were taking strictly supplements of folate and B6, and then foods that don't contain nonvitamin nutrients that inhibit heart disease, which is consistent with the stimulus. This would make (A) out of scope, which was my reasoning for eliminating it. Thoughts on this?
 
giladedelman
Thanks Received: 833
LSAT Geek
 
Posts: 619
Joined: April 04th, 2010
 
 
 

Re: Q9 - A recent study of several hundred female

by giladedelman Tue Jan 17, 2012 10:40 am

Whether the physicians in the study were taking supplements is what is in fact out of scope. Why? Because the study looks at dietary intake of folate and B6. The premise is that the more folate and B6 in your diet (in other words, the more folate and B6 you eat -- NOT the amount in your body overall, so we DON'T care about supplements), the lower your risk of heart disease and vice versa.

The problem is that you can't go from that premise to the conclusion that B6 and folate prevent heart disease because correlation does not imply causation. Maybe something else is responsible! Answer (A) provides that something else: it turns out that other nutrients in foods with folate and B6 are what inhibit heart disease. This gives us an alternate explanation.
 
jamiejames
Thanks Received: 3
Atticus Finch
Atticus Finch
 
Posts: 116
Joined: September 17th, 2011
 
 
 

Re: Q9 - A recent study of several hundred female

by jamiejames Sun Jun 03, 2012 2:45 pm

goriano Wrote:
giladedelman Wrote:I suppose they could have, but what does that have to do with the argument?


although (A) is the best answer, i dislike how this question is worded because the female physicians could have been taking supplements of folate and b6 along with foods that do not contain nonvitamin nutrients that inhibit heart disease.


I share socatkn1p's same concern with this question. The stimulus states that coronary heart disease was inversely proportional to their "dietary intake of folate and B6." To me, this means that the correlation was looking at folate and B6 IN ISOLATION, not folate, B6, plus whatever else in their diet. I think socatkn1p was addressing the possibility that the physicians were taking strictly supplements of folate and B6, and then foods that don't contain nonvitamin nutrients that inhibit heart disease, which is consistent with the stimulus. This would make (A) out of scope, which was my reasoning for eliminating it. Thoughts on this?


Dietary intake implies that these vitamins were in the food they were eating, not in a supplemental form. If it were, they would have wored the stimulus differently to indicate this. I can understand the concern for sure, but dietary in take means that these vitamins were within their diet, so they weren't only taking in these, but also vitamin c, omega 3, etc through their food. the study just happened to be about folate and B6.
 
aznriceboi17
Thanks Received: 5
Elle Woods
Elle Woods
 
Posts: 76
Joined: August 05th, 2013
 
 
trophy
Most Thankful
 

Re: Q9 - A recent study of several

by aznriceboi17 Mon Sep 02, 2013 3:50 am

Can someone tell me the reason why the LSAT test writers would want to use the term 'coronary disease' in the first sentence, and then 'heart disease' in the second sentence, when it seems like they're being used to refer to the same thing? The only effect I see is that they've introduced the potential for confusion over whether they are in fact the same thing, in which case you're not testing for logical reasoning ability, but medical trivia (ok maybe it's very common knowledge that the two terms refer to the same thing, but it's still trivia).
User avatar
 
maryadkins
Thanks Received: 641
Atticus Finch
Atticus Finch
 
Posts: 1261
Joined: March 23rd, 2011
 
This post thanked 1 time.
 
 

Re: Q9 - A recent study of several

by maryadkins Tue Sep 10, 2013 1:40 pm

aznriceboi17 Wrote:Can someone tell me the reason why the LSAT test writers would want to use the term 'coronary disease' in the first sentence, and then 'heart disease' in the second sentence, when it seems like they're being used to refer to the same thing?


I'd say in this case it's just to mix it up. You're not actually being tested on knowing that they're the same thing, as you suggest, because none of the answer choices try to catch you on that point by saying something like "the vitamins inhibit coronary disease but not heart disease." If there were an answer choice like that I'd be more inclined to agree with you, but as a general matter the LSAT isn't going to test you on trivia. Term shifts matter when they are apparent gaps in the logic of the thing. In this case, if you thought coronary and heart disease were different things, I'm not sure how that would either help or hinder you in working through the answers. They're just not about that.

Hope this helps!
 
oscey12
Thanks Received: 2
Vinny Gambini
Vinny Gambini
 
Posts: 15
Joined: August 27th, 2014
 
 
 

Re: Q9 - A recent study of several

by oscey12 Mon Oct 13, 2014 5:26 pm

I see why A is correct, but I though that D did the same thing (supplying an alternative that might be responsible). Perhaps it is not B6 and Folate, but arrhythmia (or any other heart condition) that is responsible for the development of heart disease. Any help as to why this is incorrect?
 
gaheexlee
Thanks Received: 10
Elle Woods
Elle Woods
 
Posts: 55
Joined: May 27th, 2014
 
This post thanked 1 time.
 
 

Re: Q9 - A recent study of several

by gaheexlee Thu Nov 20, 2014 1:52 pm

oscey12 Wrote: I though that D did the same thing (supplying an alternative that might be responsible).


I too was tempted by (D) so let's play out some hypotheticals and say the physicians were in fact screened for existing heart conditions.

Scenario 1: A minority/none of them had heart conditions.

Scenario 2: Half had heart conditions, half didn't.

Scenario 3: A majority/all of them had heart conditions.

Now that you have these different types of background information, let's add the premise given to us in the stimulus.

Scenario 1: A minority/none of them had heart conditions. The female physicians' tendencies to develop heart disease was inversely proportional to their intake of two particular vitamins.

Scenario 2: Half had heart conditions, half didn't. The female physicians' tendencies to develop heart disease was inversely proportional to their intake of two particular vitamins.

Scenario 3: A majority/all of them had heart conditions. The female physicians' tendencies to develop heart disease was inversely proportional to their intake of two particular vitamins.

Can you see why (D) is incorrect now? Even if we do have data on how many of the women had preexisting conditions, we still have to accept the fact that they had the tendency described above. The premise is to be applied universally, to all possible situations, because it is something we take as given.

Hope that helped!
 
anne.monjar
Thanks Received: 0
Vinny Gambini
Vinny Gambini
 
Posts: 3
Joined: January 29th, 2015
 
 
 

Re: Q9 - A recent study of several

by anne.monjar Mon Apr 20, 2015 9:04 pm

My confusion on answer A is that even if there were significant amounts of nonvitamin nutrients that inhibit heart disease in their food, that doesn't necessarily imply that folate and B6 don't inhibit heart disease. Doesn't it just mean that there might be other things that help, too?
User avatar
 
maryadkins
Thanks Received: 641
Atticus Finch
Atticus Finch
 
Posts: 1261
Joined: March 23rd, 2011
 
 
 

Re: Q9 - A recent study of several

by maryadkins Mon Apr 27, 2015 8:51 am

anne.monjar Wrote:My confusion on answer A is that even if there were significant amounts of nonvitamin nutrients that inhibit heart disease in their food, that doesn't necessarily imply that folate and B6 don't inhibit heart disease.


What it implies is that the conclusion that it's folate and B6 inhibiting the heart disease isn't a scientifically valid conclusion. Sure, they might be, still. But THIS evidence doesn't prove/show that. There are other things in the mix that could be doing the work. Therefore, it's a valid weakener.
 
dhlim3
Thanks Received: 4
Jackie Chiles
Jackie Chiles
 
Posts: 34
Joined: January 19th, 2015
 
 
 

Re: Q9 - A recent study of several

by dhlim3 Tue Mar 08, 2016 2:40 am

gaheexlee Wrote:
oscey12 Wrote: I though that D did the same thing (supplying an alternative that might be responsible).


I too was tempted by (D) so let's play out some hypotheticals and say the physicians were in fact screened for existing heart conditions.

Scenario 1: A minority/none of them had heart conditions.

Scenario 2: Half had heart conditions, half didn't.

Scenario 3: A majority/all of them had heart conditions.

Now that you have these different types of background information, let's add the premise given to us in the stimulus.

Scenario 1: A minority/none of them had heart conditions. The female physicians' tendencies to develop heart disease was inversely proportional to their intake of two particular vitamins.

Scenario 2: Half had heart conditions, half didn't. The female physicians' tendencies to develop heart disease was inversely proportional to their intake of two particular vitamins.

Scenario 3: A majority/all of them had heart conditions. The female physicians' tendencies to develop heart disease was inversely proportional to their intake of two particular vitamins.

Can you see why (D) is incorrect now? Even if we do have data on how many of the women had preexisting conditions, we still have to accept the fact that they had the tendency described above. The premise is to be applied universally, to all possible situations, because it is something we take as given.

Hope that helped!



What if the screening revealed that the proportion of those with pre-existing heart condition matched closely with the proportion of those that "developed" heart disease after the study? This would weaken the premise (which is acceptable in a weaken question - you either weaken the premise or the conclusion). I still don't understand why D is any less correct than A.
 
rhkwk1441
Thanks Received: 0
Vinny Gambini
Vinny Gambini
 
Posts: 11
Joined: December 26th, 2015
 
 
 

Re: Q9 - A recent study of several

by rhkwk1441 Tue May 31, 2016 8:36 am

I also narrowed it down to (A) and (D).
I don't know if my take on this is right or not but here's how I eliminated (D).

My problem with (D) was the use of "none." (D) basically talks about the appropriateness of the sample and I don't see any reason why the sample must be ensured that "none" has preexisting heart conditions to be appropriate. Maybe some physicians within the group had heart conditions to begin with and this has nothing to do with the argument. Maybe those with preexisting heart condition had the lowest amount of folate and B6, which goes along with the overall argument. In other words, including some physicians with preexisting heart condition does not make the sample unrepresentative or inappropriate.

If (D) said something like "all of the physicians in the study had preexisting heart conditions," this could definitely ruin the argument by making the sample severely unrepresentative. In this case, the inversely proportional relationship cannot be established because they all already possess heart conditions.

On the other hand, (A) is like a classic weakener for correlation/causation problem.
I have seen this pattern at least 10 times over my course of study.

A is correlated with B.
Therefore, A causes B.
Right answer: Wait! A comes with C and maybe C causes B.
User avatar
 
LolaC289
Thanks Received: 21
Elle Woods
Elle Woods
 
Posts: 92
Joined: January 03rd, 2018
 
 
 

Re: Q9 - A recent study of several

by LolaC289 Wed Aug 01, 2018 7:34 am

I don't feel like convinced by the way in which previous posts explained why (D) is incorrect.

Although the inverse proportional relationship is applied universal, no matter it's women with or without preexisting heart problems, if all participants do have preexisting heart problems, it may hurts the conclusion because it may be considered these participants are not representative for all women(many of whom do not have that condition).

But I do have another way to look at it though I'm not sure it is right.

Because the causation here is an "inhibition" relationship, which is to decrease or limit, it may actually strengthen the argument if the participants are troubled by heart problems before. By observing the inversely proportional relationship in them, folate & B6 can be said to be even more efficient to inhibit the development of heart disease than observing it in normal, healthy women.

Any thought?